0% found this document useful (0 votes)
47 views5 pages

Math 171, Spring 2010 Please Send Corrections To Henrya@math - Stanford.edu

This document contains solutions to homework problems from a math class. It addresses several limits and sequence problems. The key points are: - It proves that if a sequence has a positive limit L, then the limit of the sequence multiplied by a real number x is Lx. - It shows that if a function satisfies a Lipschitz condition, then a recursively defined sequence using that function is Cauchy. - It computes limits of suprema and infima for various sequences, including showing lim sup and lim inf are equal if the sequence converges.

Uploaded by

331Sachin Kiragi
Copyright
© © All Rights Reserved
We take content rights seriously. If you suspect this is your content, claim it here.
Available Formats
Download as PDF, TXT or read online on Scribd
0% found this document useful (0 votes)
47 views5 pages

Math 171, Spring 2010 Please Send Corrections To Henrya@math - Stanford.edu

This document contains solutions to homework problems from a math class. It addresses several limits and sequence problems. The key points are: - It proves that if a sequence has a positive limit L, then the limit of the sequence multiplied by a real number x is Lx. - It shows that if a function satisfies a Lipschitz condition, then a recursively defined sequence using that function is Cauchy. - It computes limits of suprema and infima for various sequences, including showing lim sup and lim inf are equal if the sequence converges.

Uploaded by

331Sachin Kiragi
Copyright
© © All Rights Reserved
We take content rights seriously. If you suspect this is your content, claim it here.
Available Formats
Download as PDF, TXT or read online on Scribd
You are on page 1/ 5

Homework 3 Solutions

Math 171, Spring 2010


Please send corrections to [email protected]

17.4. Let {an } be a sequence with positive terms such that limn→∞ an = L > 0. Let x be a real number.
Prove that limn→∞ axn = Lx .

Solution. Let  > 0. By Theorem 17.4, note that L < (Lx + )1/x and L > (Lx − )1/x . Since
limn→∞ an = L, there exists some N such that n ≥ N implies an < (Lx + )1/x and an > (Lx − )1/x .
Hence by Theorem 17.4, for n ≥ N we have axn < Lx + and axn > Lx −. This shows limn→∞ axn = Lx .

19.3. Let 0 ≤ α < 1, and let f be a function from R → R which satisfies


|f (x) − f (y)| ≤ α|x − y|
for all x, y, ∈ R. Let a1 ∈ R, and let an+1 = f (an ) for n = 1, 2, .... Prove that {an } is a Cauchy
sequence.

Solution. First we prove by induction on n that |an+1 − an | ≤ αn−1 |a2 − a1 | for all n ∈ N. The base
case n = 1 is obvious. Assuming the formula is true when n = k, we show it is true for n = k + 1:
|ak+2 − ak+1 | = |f (ak+1 ) − f (ak )| ≤ α|ak+1 − ak | ≤ ααk−1 |a2 − a1 | = αk |a2 − a1 |
Hence, by induction, this formula is true for all n.
Note that if |a2 − a1 | = 0, then an = a1 for all n, and so the sequence is clearly Cauchy. Hence we
(1−α)
6 0. Now, given any  > 0, pick N such that αN −1 < |a
consider the case when |a2 − a1 | = 2 −a1 |
, which
n
we can do because 0 ≤ α < 1 =⇒ limn→∞ α = 0. Then, for any m, n ≥ N , with m ≥ n, we have
m−1
X
|am − an | ≤ |ai+1 − ai | by the triangle inequality
i=n
m−1
X
≤ αi−1 |a2 − a1 | by our formula above
i=n
m−1
X
= |a2 − a1 | αi−1
i=n

X
≤ |a2 − a1 | αi−1
i=N
αN −1
= |a2 − a1 | by the formula for the sum of an infinite geometric series
1−α
<  by our choice of N.

Hence {an } is a Cauchy sequence.

20.6. Compute lim supn→∞ an and lim inf n→∞ an , where an is ...

Solution.
(a) n1
1
1 1 1
Since we know limn→∞ n = 0, we know lim supn→∞ n = 0 = lim inf n→∞ n by Theorem 20.4.

(b) (1 + n1 )n
Since we know limn→∞ (1 + n1 )n = e, we know lim supn→∞ (1 + n1 )n = e = lim inf n→∞ (1 + n1 )n by
Theorem 20.4.

(c) (−1)n (1 − n1 )
Since −1 ≤ (1+ n1 )n ≤ 1 for all n, we have lim inf n→∞ (1+ n1 )n ≥ −1 and lim supn→∞ (1+ n1 )n ≤ 1.
Since the subsequence {a2n−1 } has limit −1 and the subsequence {a2n } has limit 1, we have
lim inf n→∞ (1 + n1 )n ≤ −1 and lim supn→∞ (1 + n1 )n ≥ 1. Hence lim inf n→∞ (1 + n1 )n = −1 and
lim supn→∞ (1 + n1 )n = 1.

20.7. Compute lim supn→∞ an and lim inf n→∞ an and La , where a1 , a2 , . . . is an enumeration of the ra-
tional numbers in the closed interval [0, 1].

Solution. We show that La = [0, 1]. It is easy to show La ⊂ [0, 1].


To show La ⊃ [0, 1], let s ∈ [0, 1]. First we consider the case s > 0. By Theorem, 17.1, there exists
an increasing rational sequence {rn } with limit s. As s > 0, for n sufficiently large we have rn ≥ 0,
so we may assume that rn ≥ 0 for all n, hence rn ∈ [0, 1] for all n. By induction on n, we define a
sequence {bn } which is a subsequence of both {an } and {rn }. For the base case, set b1 = r1 = ak for
some integer k. For the inductive step, suppose we have defined b1 , . . . , bn and bn = rl = ak . Since
a1 , a2 , . . . is an enumeration of the rational numbers, and since the set {rl+1 , rl+1 , . . . } is infinite but
{a1 , . . . ak } is finite, there exists some k 0 > k such that ak0 = rl0 for some l0 > l. Set bn+1 = rl0 = ak0 .
Note that {bn } is a subsequence of both {an } and {rn }. Since {bn } is a subsequence of {rn }, we
have limn→∞ bn = limn→∞ rn = s. Since {bn } is a subsequence of {an }, this shows s ∈ La . The
case s = 0 is analogous. Hence La ⊃ [0, 1], so La = [0, 1].
Since La = [0, 1], we have lim supn→∞ an = lub La = 1 and lim inf n→∞ an = glb La = 0.

20.9. Let {an } be a bounded sequence such that every convergent subsequence of {an } has a limit L.
Prove that limn→∞ an = L.

Solution.
Method 1: Note that La = {L}. Hence lim supn→∞ an = lub(La ) = L = glb(La ) = lim inf n→∞ an .
So by Theorem 20.4, limn→∞ an = L.

Method 2: Suppose for a contradiction that {an } does not have limit L. Hence there exists an
 > 0 such that for any integer N , there exists some n > N with |an − L| > . This allows us to
define n1 < n2 < n3 . . . such that |ani − L| >  for all i. Since {ani } is a bounded sequence, by
Bolzano-Weierstrass it has a convergent subsequence, which clearly does not converge to L. This is
a contradiction, and so it must be that limn→∞ an = L.

20.13. Let {an } and {bn } be sequences such that {an } is convergent and {bn } is bounded. Prove that

lim sup(an + bn ) = lim sup an + lim sup bn


n→∞ n→∞ n→∞

and
lim inf (an + bn ) = lim inf an + lim inf bn
n→∞ n→∞ n→∞

Solution. Since {an } is convergent, it is bounded. So Theorem 20.6 gives us

lim sup(an + bn ) ≤ lim sup an + lim sup bn


n→∞ n→∞ n→∞
2
Let limn→∞ an = L. Note if l ∈ Lb , then l = limk→∞ bnk for some nk . Hence
lim (ank + bnk ) = lim ank + lim bnk = L + l
k→∞ k→∞ k→∞

so L + l ∈ La+b . This shows that


lim sup(an + bn ) = lub La+b ≥ L + lub Lb = lim sup an + lim sup bn
n→∞ n→∞ n→∞

We’ve shown
lim sup(an + bn ) = lim sup an + lim sup bn
n→∞ n→∞ n→∞
The proof that
lim inf (an + bn ) = lim inf an + lim inf bn
n→∞ n→∞ n→∞
is analogous.

20.20. Let {an } be a sequence of positive numbers such that limn→∞ an = L. Prove that limn→∞ (a1 a2 · · · an )1/n =
L.

Solution. Let  > 0. Then there exists some N such that n ≥ N implies an ≤ L + . Note we
have
(a1 · · · aN +m )1/(N +m) = (a1 · · · aN )1/(N +m) (aN +1 · · · aN +m )1/(N +m)
≤ (a1 · · · aN )1/(N +m) (L + )m/(N +m)
= (a1 · · · aN (L + )−N )1/(N +m) (L + )
Hence we have
lim sup(a1 a2 · · · an )1/n = lim sup(a1 · · · aN +m )1/(N +m)
n→∞ m→∞

≤ lim sup(a1 · · · aN (L + )−N )1/(N +m) (L + ) by Theorem 20.5


m→∞
=L+ by Theorem 16.4
Hence lim supn→∞ (a1 a2 · · · an )1/n ≤ L +  for all  > 0, so lim supn→∞ (a1 a2 · · · an )1/n ≤ L. Anal-
ogously, one can show that lim inf n→∞ (a1 a2 · · · an )1/n ≥ L. Hence by Theorem 20.2 and 20.4, we
have limn→∞ (a1 a2 · · · an )1/n = L.

21.2. Let An = lub{an , an+1 , . . . } and Bn = glb{an , an+1 , . . . } for n = 1, 2, . . . . Compute An , Bn ,


limn→∞ An , and limn→∞ Bn , where an =

Solution.
(a) (−1)n
Clearly An = 1 and Bn = −1 so limn→∞ An = 1 and limn→∞ Bn = −1.

(b) n1
1
Clearly An = n and Bn = 0 so limn→∞ An = 0 and limn→∞ Bn = 0.

(c) (1 + n1 )n
By Theorem 16.6 the sequence {(1 + n1 )n } is increasing and convergent with limit e. So An = e
and Bn = (1 + n1 )n so limn→∞ An = e and limn→∞ Bn = e.
(−1)n
(d) n ( (
1 −1
n n even n+1 n even
We have An = 1
and Bn = −1
. So limn→∞ An = 0 and limn→∞ Bn = 0.
n+1 n odd n n odd
(e) (−1)n (1 − n1 )
3
We have An = 1 and Bn = −1, so limn→∞ An = 1 and limn→∞ Bn = −1.
P∞ 1
22.4. Prove that the series n=1 n(n+1) converges and find its sum.

Solution. We use partial fractions to rewrite the terms. We try


1 A B
= + =⇒ 1 = (n + 1)A + nB = A + n(A + B) =⇒ A = 1 and B = −1.
n(n + 1) n n+1
1 1 1
Indeed, we have n(n+1) = n − n+1 . So the n-th partial sum is

sn = a1 + a2 + ... + an
1 1 1 1 1 1 
= − + − + ... + −
1 2 2 3 n n+1
1
=1−
n+1
This is an example of a telescoping series. Since
∞ ∞ 1 
lim sn = lim 1 − = 1,
n=1 n=1 n+1
P∞ 1
P∞ 1
we have that n=1 n(n+1) converges, and n=1 n(n+1) = 1.
P∞ P∞ P∞
23.5. Give an example of divergent series n=1 an and n=1 bn such that n=1 (an + bn ) converges.
P∞ P∞
Solution. Let an = 1 and bn = −1 for all n. Then n=1 an and n=1 bn are geometric series
with r =
P1, and hence diverge by Theorem 22.4. However, an + bn = 0 for all n so the n-th partial
∞ P∞
sum of n=1 (an + bn ) is zero for all n, giving n=1 (an + bn ) = 0 converges.
P∞
24.9. Prove that if {an } is a decreasing
P∞ sequence of positive numbers and n=1 an converges, then
limn→∞ nan = 0. Deduce that n=1 n1s diverges if 0 ≤ s ≤ 1.
Pn
Solution. Let  > 0. Let sn = k=1 ak . Since {sn } converges, {sn } is Cauchy so there ex-
ists some N such that n, m ≥ N implies |sn − sm | < . In particular, for n ≥ N we have
(n − N )an ≤ aN +1 + · · · + an = |sn − sN | < . Hence limn→∞ (n − N )an = 0. By Theorem
22.3, we have limn→∞ N an = N limn→∞ an = 0 too. So by Theorem 12.2, we have
lim nan = lim (n − N )an + lim N an = 0
n→∞ n→∞ n→∞

so we’re done.
Now, note that for 0 ≤ s ≤ 1, we have limn→∞ n n1s = limn→∞ n1−s 6= 0. By the above, this
P∞
means that n=1 n1s cannot converge, that is, it diverges.

25.2. Let {an } satisfy the hypotheses


P∞ of the alternating series test. Let {sn } denote the sequence of par-
tial sums of the series n=1 (−1)n+1 an . Prove that the sequence {s2n−1 } is decreasing and bounded
below by 0.

Solution. Note s2(n+1)−1 = s2n−1 − a2n + a2n+1 ≤ s2n−1 since a2n+1 ≤ a2n . Hence the sequence
{s2n−1 } is decreasing.
Note s2n−1 = (a1 − a2 ) + (a3 − a4 ) + · · · + (a2n−3 − a2n−2 ) + a2n−1 , where each term in parenthesis
is bounded below by 0, and a2n−1 is also bounded below by 0. Hence {s2n−1 } is bounded below by 0.

Give an example of a sequence {an } of positive numbers such that limn→∞ an = 0, but the series
25.4. P
∞ n+1
n=1 (−1) an diverges.

4
(
2
n odd
Solution. Let an = n+1 . Then limn→∞ an = 0 but the (2n − 1)-th partial sum of
0 n even
P∞ P∞ P∞
n=1P(−1)n+1 an is equal to the n-th partial sum of n=1 n1 . Since n=1 n1 diverges by Corollary

24.3, n=1 (−1)n+1 an also diverges.
P∞ P∞
26.4. Prove that if n=1 an converges absolutely, then n=1 a2n converges.

Solution. By Theorem 22.3, we have limn→∞ |an | = 0. So by Theorem 13.2, {|an |} is bounded.
P∞ {an } is P
Hence bounded. We apply part (i) of Theorem 26.4 (replacing {bn } with
∞ 2
P∞{an })
2
to see that
a
n=1 n na = a
n=1 n converges absolutely. Since all terms are non-negative, n=1 n converges.
a
an+1 1/n
26.8. Let {an } be a sequence of positive numbers. Prove that if limn→∞ an = L then limn→∞ an = L.
Deduce limn→∞ (n!)n1/n = e.

Solution. Define the sequence {bn } by b1 = a1 and bn = an−1 an


for n ≥ 2. Since limn→∞ aan+1
n
= L,
we have limn→∞ bn = L. Note that an = b1 b2 · · · bn . Applying exercise 20.20 to the sequence {bn },
we get
lim an1/n = lim (b1 b2 · · · bn )1/n = L
n→∞ n→∞

nn
Now, let an = n! . Note that
(n+1)n+1 (n+1)n+1
an+1 (n+1)! n+1 n+1 n 1
lim = lim nn = lim = lim ( ) = lim (1 + )n = e
n→∞ an n→∞
n!
n→∞ nn n→∞ n n→∞ n
By the conclusion above, we have
n
lim = lim a1/n
n =e
n→∞ (n!)1/n n→∞

You might also like